what is x + 3 1/2 = 9

Answers

Answer 1

Answer:

Step-by-step explanation:

x+3 1/2=9

x=9-7/2

x=(18-7)/2=11/2

x=5 1/2

Answer 2

Answer:

3 1/2 + 5 1/2= 9.

Step-by-step explanation:

Hope this helps? im sorry if it doesn't :)


Related Questions

Write the vertex form equation of a parabola that has been translated right of the origin by 5 units, translated down from the origin by 15 units, reflected over the x-axis, and stretched vertically by a factor of
6

Answers

Answer:

90x+8202727

Step-by-step explanation:

90x70+8090#* 77

A square plot is 14 meters long.Find its area?​

Answers

Answer:

Area of square= side*side

So we will have,

14*14

= 196cm²

The Coners family are going on a 250 mile road trip. The first day they drive 100 miles. Write an equation with a variable that can be used to find the number of miles remaining.​

Answers

Answer:

100+x=250 or 250-x=100

Step-by-step explanation:

Item 3
Use the Distributive Property to simplify the expression.

15 (4n-2)

Answers

Answer:

distributive property solve 3 into 4 - 2

Hello!

The Distributive Property states the following:

a(b+c)=ab+ac

where

a, b, and c can be either constants or variables.

Now, simplify:

15(4n-2)

15*4n - 15*2

60n-30

And we can't simplify it any further, because 60 n and 30 aren't like terms.

Hope everything is clear.

Let me know if you have any questions!

#KeepOnLearning

:-)

I need help fast i apericiate it my guy

Answers

Answer: 59

Step-by-step explanation:

Answer:

I think it's letter b.29

pythagorean theorem converse please help i’ll mark brainlest

Answers

Answer:

thanks

Step-by-step explanation:

We can prove this by contradiction.

Let us assume that c2=a2+b2 in ΔABC and the triangle is not a right triangle.

Now consider another triangle ΔPQR. We construct ΔPQR so that PR=a, QR=b and ∠R is a right angle.

By the Pythagorean Theorem, (PQ)2=a2+b2.

But we know that a2+b2=c2 and a2+b2=c2 and c=AB.

So, (PQ)2=a2+b2=(AB)2.

That is, (PQ)2=(AB)2.

Since PQ and AB are lengths of sides, we can take positive square roots.

PQ=AB

That is, all the three sides of ΔPQR are congruent to the three sides of ΔABC. So, the two triangles are congruent by the Side-Side-Side Congruence Property.

Since ΔABC is congruent to ΔPQR and ΔPQR is a right triangle, ΔABC must also be a right triangle.

This is a contradiction. Therefore, our assumption must be wrong.

Step-by-step explanation:

We can prove this by contradiction.

Let us assume that c2=a2+b2 in ΔABC and the triangle is not a right triangle.

Now consider another triangle ΔPQR. We construct ΔPQR so that PR=a, QR=b and ∠R is a right angle.

By the Pythagorean Theorem, (PQ)2=a2+b2.

But we know that a2+b2=c2 and a2+b2=c2 and c=AB.

So, (PQ)2=a2+b2=(AB)2.

That is, (PQ)2=(AB)2.

Since PQ and AB are lengths of sides, we can take positive square roots.

PQ=AB

That is, all the three sides of ΔPQR are congruent to the three sides of ΔABC. So, the two triangles are congruent by the Side-Side-Side Congruence Property.

Since ΔABC is congruent to ΔPQR and ΔPQR is a right triangle, ΔABC must also be a right triangle.

This is a contradiction. Therefore, our assumption must be wrong

Rhombus
b= 14 in.
h=9 in.


Paralelogram
b= 12ft
h= 8.5ft


A rhombus has an area of 375 mm2 and a base of 25 mm. What is its height?

Answers

Answer:

Step-by-step explanation:

The solution is, The area of Rhombus B is 9 times bigger than the area of Rhombus A.  The reason is that 2 dimensions were tripled, so x3 x3 = 9 times bigger.

What is area ?

Area is the measure of a region's size on a surface. The area of a plane region or plane area refers to the area of a shape or planar lamina, while surface area refers to the area of an open surface or the boundary of a three-dimensional object.

here, we have,

Rhombus A:

Area = 35 square inches

base = 7 inches

height = 5 inches

Rhombus B:

Area = 315 square inches

base = 21 inches (7 x 3)

height = 15 inches (5 x 3)

A = 21 x 15

so, we get,

The area of Rhombus B is 9 times bigger than the area of Rhombus A.  The reason is that 2 dimensions were tripled, so x3 x3 = 9 times bigger.

To learn more on Area click:

brainly.com/question/20693059

#SPJ2

complete question:

Rhombus A has base of 7 inches and an area of 35 square inches. Rhombus B has a Base and Height that are three times the base and height of Rhombus A. Find the area of Rhombus b and compare it to the area of rhombus A Explain

Does anyone know?? I'll give a brainliest For the correct answer.​

Answers

Answer:

its 14

Step-by-step explanation:

because you have 40 bucks then you minus 18 then you divide then u get 14.

Answer:

$14

Step-by-step explanation:

20*2=40

40-12=28

28/2=14

This graph shows the water temperature as a function of time.

Answers

Answer:

ioaiaifzkziz%syzjzxsbshshbsndnxjddd

For the quadratic function shown below, find (a) the direction of the opening, (b) the vertex, and (c) the axis of symmetry.
y=-3(x-3)^2+5

Answers

Answer:

honestly I don't know the answer

A. Find the product. Rewrite +
5.67 X 1.25

Answers

Answer:

Multiply

5.67 by 1.25.

7.0875

What is the expression that means the same as 7 less than the square of a number? A. 7-2x B. 7-2× C. 7-x2 D. x2-7 ​

Answers

answer d is correct!!!!!

Answer:

D

Step-by-step explanation:

7 less than the square of the number means a number squared subtracted by 7, which would be D.

You invested $7000 in two accounts paying 7% and 8% annual interest, respectively. If the total interest earned for the year was $536, how much was invested at each rate?

Answers

Answer: $4,600 was invested in the 8% account and $2,400 was invested in the 7% account.

Step-by-step explanation:

Let's break this down into simpler terms so you can understand.

You invested $7000 in a 7% and 8% account (always remember: the 'x' variable is always given to the biggest percentage)

Total of interest: 536 (this always goes at the end of the equation, after the equal sign)

Step 1: Let's set up the equation:

Here are how percents are written: (7% = 0.07, 8% = 0.08, other examples: 50% = 0.50)

0.08x + 0.07(7,000-x)=536

You're adding both because you want to see how much each account paid together.

Step 2: Distribute the 0.07 to everything in the parenthesis

0.08x + 0.07(7,000-x)=536

0.08x + 490 - 0.07x =536

Step 3: Combine like terms

0.08x + 490 - 0.07x =536

0.01x + 490 =536

Step 4: Continue combining like terms

0.01x + 490 =536

0.01x = 46

Step 5: Divide both sides by 0.01 to get the 'x' by itself

0.01x = 46

x = 4,600

This answer, x = 4,600, says to us that $4,600 was invested in the 8% account.

To find out how much was invested in the 7% account, just replace the x = 4,600 back into this part of the original equation: 7,000-x

7,000-x

7,000-4,600

2,400

So.. $4,600 was invested in the 8% account and $2,400 was invested in the 7% account.

HELPPPPPPPP!!!!!!!!!!!!!!!!!!!!

Answers

Answer:

4

Step-by-step explanation:

- 14 + 30 / 4

16 / 4

16 ÷ 4

= 4

^-^

Answer:

4

Step-by-step explanation:

-2×7=-14

-2×15=-30

-14--30=16

16÷4=4

6 less than the number y

Answers

Answer:

6 less a number y can be represented by the expression 6 - y

Solve the system using substitution. Show your work and give your answers as an ordered pair.​

Answers

Solution and answer below.

What is 56x1
Just a regular normal question from me
Your welcome in advance

Answers

Answer:

56

Step-by-step explanation:

hope this helps

plz give me brailiest

Answer:

Step-by-step explanation:

5+4=24

1) If Donkey Kong is on a bridge 638 feet in the air, 6 seconds after a shell
was released, which shell hit Donkey Kong and how do you know?
+

Answers

Answer:

5000inches

Step-by-step explanation:

i think is that!

Find the solution to the system of equations.
You can use the interactive graph below to find the
solution.
–2x + 2y = -4
3x + 3y = -18

Answers

No solution it’s infinite

If I have a ratio 2:6, then what is x/18?
f I have a ratio 5:7, then what is (x+3):7?

Answers

Answer:

1. x=6

2. x=2

Step-by-step explanation:

2:6

x:18

From 6 to 18, 6 was multiplied by 3. To keep the equality, we must multiply the other side by three as well. Therefore, x is equal to 2*3 which equals 6.

x=6.

5:7

(x+3):7

For this one, the second value did not change, so we know that x+3 has to equal 5. Therefore, x=2.

x=2

Factor Completely'''​

Answers

Answer:

[tex]4x^2(x-2)(x+6)[/tex]

Step-by-step explanation:

Factor [tex]4x^2[/tex]

[tex]4x^2(x^2)+4x^2(4x)-4x^2(-12)[/tex]

[tex]4x^2(x^2+4x-12)[/tex]

[tex]4x^2(x-2)(x+6)[/tex]

good luck, hope this helps :)

true or false please help thank you

Answers

Answer:

True.

See below for explanation.

Step-by-step explanation:

Reason 1

When we solve the equation by combining both equations in one, the variable will become 0.

Proof

[tex] \begin{cases} y = 4x - 3 \\ y = 4x - 5 \end{cases} \\ 4x - 3 = 4x - 5 [/tex]

Combine like terms.

[tex]4x - 4x = - 5 + 3 \\ 0 = - 2[/tex]

See that the equation is false no matter which numbers you substitute in x-term.

Reason 2

Both graphs are parallel and don't have any intercepts to each others.

See the attachment below for graphs.

Since both graphs do not really have intercepts, that means they do not provide any solutions. Hence, no solutions.

The cost of making copies at Store W is ​$6 regardless of the number of copies. The cost of making copies at Store Z is 50¢ per copy plus a ​$1 charge for the use of the machine. The total​ cost, c, of making n copies at either store can be represented by an equation. Write and graph a system to find how many copies you have to make for the cost to be the same at both stores. Use pencil and paper. If you have to make a small number of copies which store should you go​ to? If you have to make a large number of copies which store should you go​ to? Explain.
Write the system of equations. Choose the correct answer below.
A.
Store W
c=6
Store Z
c=0.50n−1
B.
Store W
c=0.50n−6
Store Z
c=1
C.
Store W
c=0.50n+1
Store Z
c=6
D.
Store W
c=6
Store Z
c=0.50n+1
Your answer is correct.
Graph the system of equations. Choose the correct answer below





(Anwser fast please)

Answers

i know this one give me a second

What's 2/5 divided by 1/10?
Please help

Answers

Answer:

4

Step-by-step explanation:

That's the answer......

Answer:

2/5 ÷ 1/10 = 4

Hope this helped!

a taxi charges $2 per mile and $1 per person. If a ride for 2 people costs $12, how many miles was the ride?

Answers

Answer:

5 miles

Step-by-step explanation:

$12 = ($2 per mile) (2 people)

$12 = $2 + $2m (m = mile)

subtract 2 on both sides to eliminate

$12 - $2 = $2 - $2 + 2m

$10 = 2m

divide both sides by 2 to eleminate m

$10 / 2 = 2m / 2

m = 5

3.3: Boiling Water


The function W gives the temperature, in degrees Fahrenheit, of a pot of water on a stove,


1 minutes after the stove is turned on.


1. Take turns with your partner to explain the meaning of each statement in this


situation. When it's your partner's turn, listen carefully to their interpretation. If you


disagree, discuss your thinking and work to reach an agreement.


a. W(0) = 72


b. W(5) > W(2)


dua 000


Sold


c. W(10) = 212wingsteam to bilo na


Darsning


d. W(12) = W(10)


e. W(15) > W(30)


f. W(0)

Answers

Answer:

Initial temperature, temperature at time 0

Step-by-step explanation:

The temperature of water in degree Fahrenheit 1 minute after stove is turned on ;

W(0) = 72 ; Initial temperature od water, that is temperature when time, t = 0

W(5) > W(2) ; This will be true, the tperature after 5 minutes should be greater Than the temperature after 2 minutes

W(10) = 212 ; The temperature of water is 212 degree Fahrenheit after 10 minutes

W(12) = W(10)

e. W(15) > W(30)

The ratio of plates to cups is 10:14. If
there are 7 cups, how many plates
are there?
(Help)

Answers

x PLATES ... 10

7 CUPS ... 14

Cross multiply and et' violá:

[tex]14x=70\implies x=70/14=5[/tex]

10 % of what number is 29?

Answers

Answer:

290

Step-by-step explanation:

To solve this problem you multiply 29 by 100 and then divide the total by 10 as follows:

(29 x 100) / 10

Therefore, you can derive that 10 percent of 290 equals 29.

Note that the answer provided is rounded to the nearest two decimals if necessary.

Kimberly has $120 to spend at the bookstore. Kimberly buys a hardcover book for $36, as well as some gift cards for her family and friends. Each gift card is $15. How many gift cards can Kimberly buy

Answers

Answer:

she can buy 5 more

Step-by-step explanation:

5.6 but she can’t buy half a gift card so 5 gift cards is your answer :)

2 3/4 + 5 1/2

PLEASE I NEED ANSWER URGENT

Answers

the answer is 8 1/4!!
Answer:
The answer is 8 1/4
Other Questions
Plz help fast will mark brainliest What did Morrie encourage Mitch to do his senior year? What do you think Carranza means by "additional trauma?" The diagram shows forces acting on a boat.Which arrow represents the direction of upthrust?look at the photo find the finance charge for a $25,000, three year loan with a 9.25% apr. How much interest is paid Is VUW similar toVXY? If yes,explain how you know. If no, explainwhy not. hi, i need this quickly, thanks so much summary on listening to music while studying Que es una Progresin Aritmticas What does Mercutio's speech about Queen Mab reveal about his character?He is always happy and likes to please other people.He does not know many people at the party that night.He wants to go home instead of going out to a party.He has a temper and is annoyed by lovers and dreamers. What characteristics designate a school as HBCU? How might the college experience be different at an HBCU? How might it be the same? inequalities!! please help!! help me iv been trying from this morning Find the next two terms of each geometric sequence (show work)1. -15, -30, -6, __, __2. 90, 30, 10, __, __3. 1/4, 3/8, 9/16, __, __ You have 2 resistors of unknown values you label Ra and Rb. You have an old battery and a multimeter you bought years ago for 7$ at Harbor Freight. Using the meter in voltage mode, you measure 10 V across the battery. You then connect the 2 resistors in series across the battery and use the meter in current mode to find the current flowing through the circuit. It reads 0.111A. You then connect the 2 resistors in parallel across the battery and use the meter again to measure the current now coming from the battery to be 0.5A. With this information you have gathered, you find the value of the 2 resistors.Value of smallest resistance in ohms. a. 60 b. 90 c. 20 d. 30 How to make 10.74 in two different ways? Solve each problem. Show all work.1. If the mass of an object is 5 kg and its velocity is 7.3 m/s, calculate the kineticenergy What was the impact of the Cuban revolution? For the triangle it is given that AE2 + EB2 = 9 and BE2 + EC2 = 16 Find AC = What must be on the blank to make statement true.. (3x+_)(xx-4)=3x-11x-4?A.1B.-bC.13 D. - 13Pls help me